Đến nội dung

Kim Vu nội dung

Có 145 mục bởi Kim Vu (Tìm giới hạn từ 28-04-2020)



Sắp theo                Sắp xếp  

#718666 Chứng minh $\prod_{j=1}^{p} (j^2+1)$ chia...

Đã gửi bởi Kim Vu on 24-12-2018 - 21:14 trong Số học

Cho số nguyên tố p>3. Chứng minh số dư của phép chia $\prod_{j=1}^{p} (j^2+1)$ cho $p$ là 0 hoặc 4.

Xem tại ĐÂY




#716325 Tính $\lim_{n\rightarrow+\infty} \frac...

Đã gửi bởi Kim Vu on 06-10-2018 - 17:45 trong Dãy số - Giới hạn

Cho dãy số $(x_n)$ xác định bởi: $x_0=2018, x_n=\frac{-2018}{n} \sum_{i=0}^{n-1} x_i (\vee n \geq 1)$. Tính $\lim_{n\rightarrow+\infty} \frac{n^2.\sum_{i=0}^{2018}2^i.x_i+5}{-2019n^2+4n-3}$

Xét $n\leq 2018$
$x_n=\frac{-2018}{n} \sum_{i=0}^{n-1} x_i$

$\Rightarrow  x_{n-1}=\frac{-2018}{n-1} \sum_{i=0}^{n-2} x_i$

$\Rightarrow nx_{n}-(n-1)x_{n-1}=-2018x_{n-1}$

$\Rightarrow x_{n}=\frac{n-2019}{n}x_{n-1}$

$\Rightarrow x_{n}=\frac{n-2019}{n}.\frac{n-2020}{n-1}x_{n-2}$

$=...=x_{0}\frac{(n-2019)(n-2020)...-(2018)}{n(n-1)...1}$

$=x_{0}.\frac{(-1)^n.2018!}{n!.(2018-n)!}$

$=x_{0}.(-1)^nC_{2018}^{n}$

Do đó $\sum_{i=0}^{2018}2^ix_{i}=-2018\sum_{i=0}^{2018}(-1)^iC_{2018}^{i}.2^i=-2018(1-2)^{2018}=-2018$

Vậy $\lim_{n\rightarrow+\infty} \frac{n^2.\sum_{i=0}^{2018}2^i.x_i+5}{-2019n^2+4n-3}=\lim_{n\rightarrow+\infty} \frac{n^2.(-2018)+5}{-2019n^2+4n-3}=\frac{2018}{2019}$

 




#716220 Tìm số dư của phép chia

Đã gửi bởi Kim Vu on 01-10-2018 - 23:01 trong Số học

$A=\prod_{i=1}^{p-1}(i^2+1)$
$\equiv \prod_{i=1}^{\frac{p-1}{2}}(i^2+1)^2$(do $i^2 \equiv (p-i)^2(mod p),\forall i=1,\frac{p-1}{2}$)
Xét đa thức $P(x)=\prod_{i=1}^{\frac{p-1}{2}}(x-i^2)-(x^{\frac{p-1}{2}}-1)$
Đa thức trên có bậc $<\frac{p-1}{2}$ và phương trình $P(x) \equiv 0(mod p)$ có $\frac{p-1}{2}$ nghiệm không đồng dư modulo $p$ là $1^2,2^2,...,(\frac{p-1}{2})^2$ nên theo định lí Lagrange đa thức trên có tất cả các hệ số đều chia hết cho $p$
Do đó $P(-1) \equiv 0$(mod p)
$\Rightarrow \prod_{i=1}^{\frac{p-1}{2}}(i^2+1) \equiv 2$ (mod p)
$\Rightarrow A \equiv 4$(mod p)

 




#716162 Cho đường tròn $(O)$ các cạnh của tam giác $ABC$ tại sáu...

Đã gửi bởi Kim Vu on 30-09-2018 - 20:12 trong Hình học phẳng

Cho đường tròn $(O)$ các cạnh của tam giác $ABC$ tại sáu điểm phân biệt $D,E,F,G,I,H$. sao cho $D$ và $E$ nằm trên $BC$, $F$ và $G$ nằm trên $CA$, $I$ và $H$ nằm trên $AB$. Chứng minh rằng nếu các đường thẳng đi qua $D$ vuông góc $BC$, qua $F$ vuông góc $CA$, qua $H$ vuông góc $AB$ đồng quy thì các đường thẳng đi qua $E$ vuông góc $BC$, qua $G$ vuông góc $CA$, qua $I$ vuông góc $AB$ đồng quy.

Bổ đề đẳng giác:Cho $P$ và $Q$ là hai điểm liên hợp đẳng giác đối với tam giác $ABC$. Gọi $X, Y, Z$ lần lượt là các hình chiếu của $P$ trên các cạnh $BC, AC, AB$ và $L , M , N$ lần lượt là các hình chiếu của $Q$ trên các cạnh $BC, AC, AB$. Khi đó, sáu điểm $X, Y, Z, L,M,N$ cùng nằm trên một đường tròn.
Bài toán trên được suy ra trực tiếp từ bổ đề này




#716044 CMR (ABG) đi qua trung điểm BF

Đã gửi bởi Kim Vu on 26-09-2018 - 22:51 trong Hình học

Cho tam giác ABC, BC>AB, E và F là điểm giữa của cung lớn AC và cung nhỏ AC. $G \in BC$ sao cho $EG \perp BC$.CMR (ABG) đi qua trung điểm BF

$(O)$ là đường tròn ngoại tiếp tam giác $ABC$
Kẻ đường kính $BL$ của $(O)$ suy ra $\widehat{BAL}=90^{\circ}$
$EG \cap (ABG)=Q \Rightarrow \widehat{BQG}=\widehat{BAG}=90^{\circ}$
Do đó $A,Q,L$ thẳng hàng
$EG\cap (O)=X$
Dễ thấy $LC \parallel EX$ nên cung $LX$=cung $EC$=cung $EA$ 
$\Rightarrow \widehat{QAX}=\widehat{QXA} \Rightarrow QA=QX \Rightarrow QO \perp AX$ 
Lại có $AX \perp BE$ do $\widehat{AXE}=\widehat{EBC}$ nên $QO \parallel BE$
Mặt khác do $\widehat{PQX}=\widehat{PBG}=\widehat{PBE}-\widehat{EBC}=90^{\circ}-\widehat{EBC}=\widehat{BEG}$ nên $PQ \parallel BE $
Do đó ta có $P,O,Q$ thẳng hàng suy ra $\widehat{QPB}=\widehat{QGB}=90^{\circ} \Rightarrow OP\perp BF \Rightarrow$ P là trung điểm $BF$

geo.png
 




#716014 $f(x^{n+1}+y^{n+1})=x^{n}f(x)+y^{n...

Đã gửi bởi Kim Vu on 25-09-2018 - 21:23 trong Phương trình hàm

Cho số nguyên dương $n$.Tìm tất cả các hàm số $f:(1,+\infty)\rightarrow\mathbb{R}$ thỏa mãn:
$f(x^{n+1}+y^{n+1})=x^{n}f(x)+y^{n}f(y),\forall x,y>1$

Spoiler




#715993 Chứng minh 3 đường thẳng đồng quy

Đã gửi bởi Kim Vu on 24-09-2018 - 23:41 trong Hình học phẳng

a)Định lí: $M$ nằm trên đoạn $AB$,$O$ bất kì ta đều có:$\overrightarrow{OM}=\dfrac{MB}{AB}.\overrightarrow{OA}+\dfrac{MA}{AB}\overrightarrow{OB}$
Gọi $G$ là trọng tâm $\triangle ABC$
Ta sẽ chứng minh $AA_{2},BB_{2},CC_{2}$ đồng quy tại $G$
Theo định lí,$\overrightarrow{AA_{2}}=\frac{A_{2}B_{1}}{C_{1}B_{1}}.\overrightarrow{AC_{1}}+\frac{A_{2}C_{1}}{C_{1}B_{1}}.\overrightarrow{AB_{1}}
=\frac{A_{2}B_{1}}{C_{1}B_{1}}.\frac{AC_{1}}{AB}.\overrightarrow{AB}+\frac{A_{2}C_{1}}{C_{1}B_{1}}.\frac{AB_{1}}{AC}.\overrightarrow{AC}=\frac{k}{k+1}.\frac{1}{k+1}.(\overrightarrow{AB}+\overrightarrow{AC})=\frac{k}{(k+1)^2}.3\overrightarrow{AG}$
Suy ra $A,A_{2},G$ thẳng hàng
Tương tự $B,B_{2},G$ thẳng hàng và $C,C_{2},G$ thẳng hàng nên ta có ĐPCM
b) Theo câu a
$\overrightarrow{AA_{2}}=\frac{k}{(k+1)^2}.3\overrightarrow{AG}$
nên $\frac{AA_{2}}{AG}=\frac{3k}{(k+1)^2}$
Tương tự suy ra $\frac{AA_{2}}{AG}=\frac{BB_{2}}{BG}=\frac{CC_{2}}{CG}=\frac{3k}{(k+1)^2}$
Do đó $\triangle A_{2}B_{2}C_{2} \sim \triangle ABC$ với tỉ số $1-\frac{3k}{(k+1)^2}$
$S_{A_{2}B_{2}C_{2}} $ min $\Leftrightarrow \frac{3k}{(k+1)^2}$ max
Đến đây áp dụng $Cauchy$ ta có $\frac{3k}{(k+1)^2} \leq \frac{3k}{4k}=\frac{3}{4}$
ĐTXR khi $k=1$
Vậy $S_{A_{2}B_{2}C_{2}} $min $\Leftrightarrow k=1$

Hình gửi kèm

  • Geo.png



#715971 Kì thi chọn HSG THPT và chọn đội tuyển thi HSG Quốc Gia tỉnh Vĩnh Long năm h...

Đã gửi bởi Kim Vu on 24-09-2018 - 17:37 trong Thi HSG cấp Tỉnh, Thành phố. Olympic 30-4. Đề thi và kiểm tra đội tuyển các cấp.

Bài 4
Đường đối trung của $\triangle ABC$ cắt $(ABC)$ tại $T$;$AN \cap (ABC)=S;PN\cap AC=V$
$L$ là điểm đối xứng $N$ qua $M$
Dễ suy ra $T$ là tâm vị tự quay biến $P \rightarrow B,V\rightarrow C,Q \rightarrow L,R \rightarrow S,$

nên $\widehat{PRQ}=\widehat{BSL}$
Mặt khác do tính đối xứng nên $\widehat{BSL}=\widehat{NSC}=\widehat{ABC}$
Suy ra $\widehat{PRQ}=\widehat{ABC} (1)$
Gọi $ QR \cap BC=U$
Từ $(1)$ suy ra tứ giác $BPRU$ nội tiếp nên $\widehat{BRU}=90^{\circ} \Rightarrow QR \perp BC$
VL.png




#715939 Kì thi chọn HSG THPT và chọn đội tuyển thi HSG Quốc Gia tỉnh Vĩnh Long năm h...

Đã gửi bởi Kim Vu on 23-09-2018 - 22:43 trong Thi HSG cấp Tỉnh, Thành phố. Olympic 30-4. Đề thi và kiểm tra đội tuyển các cấp.

42408252_714095835615584_585918101497000

Nguồn:facebook

Bài 2
Quy nạp:$u_{n}>0;u_{n+1}>u_{n},\forall n\in \mathbb{N^{*}}$
Giả sử $(u_{n})$ bị chặn trên thì tồn tại $limu_{n}=a\geq 0$
Chuyển qua giới hạn suy ra $a=-1$ mâu thuẫn
Suy ra$u_{n} \rightarrow +\infty$
Ta có:
$u_{n+1}-u_{n}=(u_{n}+1)^2;u_{n+1}+1=(u_{n}+1)(u_{n}+2)\Rightarrow \frac{1}{u_{n}+2}=\frac{u_{n+1}-u_{n}}{u_{n+1}u_{n}}=\frac{1}{u_{n}}-\frac{1}{u_{n+1}}$
Do đó $v_{n}=1-\frac{1}{u_{n+1}}$ suy ra lim $v_{n}=1$
Bài 3
a)Gọi 100 số tự nhiên là $x_{1},x_{2}...x_{100}$
$x_{1}+x_{2}+...+x_{100}=200;x_{1},x_{2}...x_{100} \leq 100$
TH1: $x_{1}=x_{2}=...=x_{100}=2$ ta có ngay ĐPCM
TH2: Tồn tại $i,j:x_{i} \neq x_{j}$ 
Giả sử $x_{1} \neq x_{2}$
Xét dãy $x_{1},x_{2},x_{1}+x_{2},x_{1}+x_{2}+x_{3},...,x_{1}+x_{2}+...+x_{99}$ chứa các số hạng đều nhỏ hơn 200
Nếu có ít nhất 1 số trong dãy chia hết cho 100 thì số đó phải bằng 100 và đó là tổng cần tìm
Nếu mọi số hạng của dãy đều không chia hết cho 100
Dãy chứa 100 số mà chia 100 dư 1,2,..,99 nên theo nguyên lí Dirichlet tồn tại 2 số hạng của dãy đồng dư với nhau theo mod 100
 $x_{1} \neq x_{2}$ và $x_{1},x_{2} \leq 100$ nên $x_{1}\not\equiv x_{2}(mod 100)$
Do đó 2 số hạng của dãy đồng dư mod 100 thì hiệu của chúng phải bằng 100 và chính là tổng cần tìm
b)$x^3y-x^3-1=2x^2+2x+y$
$\Rightarrow y(x^3-1)=x^3+2x^2+2x+1$

$\Rightarrow x^3-1\mid x^3+2x^2+2x+1$
$\Rightarrow  x^3-1 \mid x^3-1+2(x^2+x+1)$
$\Rightarrow x^3-1 \mid 2(x^2+x+1)$
$\Rightarrow x-1 \mid 2$
Từ đây tìm ra $x,y$

 




#715910 CMR $(x_n)$ có GHHH và tìm giới hạn đó

Đã gửi bởi Kim Vu on 23-09-2018 - 15:51 trong Dãy số - Giới hạn

Cho dãy số $x_1=2, x_{n+1}=\frac{n}{2n+1}(x_n+1)$. CMR $(x_n)$ có GHHH và tìm giới hạn đó

Bổ đề : Cho 2 dãy số dương $(a_{n});(b_{n})$ thỏa mãn $a_{n+1} \leq q a_{n} +b_{n}$ trong đó $q\in [0;1]$ và 
$(b_{n})$ bị chặn trên bởi 1 ; $lim b_{n}=0$
Khi đó $lim a_{n}=0$
Chứng minh bổ đề:
Từ $a_{n+1} \leq q a_{n} +b_{n}
\Rightarrow a_{n+1}\leq q^{n-1}a_{1}+b_{n}+qb_{n-1}+...+q^{n-1}b_{1}$
Do $lim  b_{n}=0$ nên tồn tại $n_{0} \in \mathbb{N^{*}}$ sao cho $\forall n\geq n_{0}, b_{n} \leq \epsilon (1-q)$
$\Rightarrow a_{n+1} \leq q^{n-1}a_{1}+\epsilon (1-q)(1+q+q^2+...+q^{n-n_{0}})+q^{n-n_{0}+1}b_{n_{0}-1}+...+q^{n-1}b_{1}$
$=q^{n-1}a_{1}+\epsilon (1-q).\frac{1-q^{n-n_{0}+1}}{1-q}+q^{n-n_{0}+1}b_{n_{0}-1}+...+q^{n-1}b_{1}$
$\leq q^{n-1}a_{1}+\epsilon(1-q^{n-n_{0}+1})+q^{n-n_{0}+1}+...+q^{n-1}$(do $b_{n} <1,\forall n\in N)$
$\Rightarrow a_{n+1} \leq q^na_{1}+\epsilon(1-q^{n-n_{0}+1})+q^{n-n_{0}}.\frac{1-q^{n_{0}}}{1-q}$
Cho $n \rightarrow +\infty,\epsilon \rightarrow 0$ ta có $lima_{n}=0$
Vậy bổ đề đã được chứng minh
Trở lại bài toán $x_{n+1}=\frac{n}{2n+1}(x_n+1)
\Rightarrow x_{n+1}-1=\frac{n}{2n+1}(x_{n}-1)-\frac{1}{2n+1} 
\Rightarrow \left |x_{n+1}-1  \right |=\left |\frac{n}{2n+1}(x_{n}-1)-\frac{1}{2n+1}   \right | \leq \frac{1}{2}\left |(x_{n}-1)  \right | +\frac{1}{2n+1}$
Đến đây áp dụng bổ đề ta có $lim \left |x_{n}-1  \right |$=0 nên $limx_{n}=1$




#715908 TÌM X, Y NGUYÊN DƯƠNG THỎA MÃN: $\sqrt{X}+\sqrt...

Đã gửi bởi Kim Vu on 23-09-2018 - 14:27 trong Số học

TÌM X, Y NGUYÊN DƯƠNG THỎA MÃN: $\sqrt{X}+\sqrt{Y}=\sqrt{X+Y}+2$

$\sqrt{x}+\sqrt{y}=\sqrt{x+y}+2(1)$
$\Leftrightarrow x+y+2\sqrt{xy}=x+y+4\sqrt{x+y}+4$
$\Leftrightarrow \sqrt{xy}=2\sqrt{x+y}+2(2)$
$\Leftrightarrow xy=4(x+y+4)+16\sqrt{x+y}$
$\Rightarrow \sqrt{x+y} \in \mathbb{Q}(3)$
$(1) \Leftrightarrow \sqrt{x}=\sqrt{x+y}+2-\sqrt{y}
\Leftrightarrow x=x+y+4+y+4\sqrt{x+y}-2\sqrt{y}(\sqrt{x+y}+2)$
 Kết hợp với $(3) \Rightarrow \sqrt{y} \in \mathbb{Q}$
Mặt khác từ $(1),(3) \Rightarrow \sqrt{x}+\sqrt{y} \in \mathbb{Q}$
Suy ra $ \sqrt{x} \in \mathbb{Q} , \sqrt{y} \in \mathbb{Q}$
Đặt $\sqrt{x}=\frac{m}{n};\sqrt{y}=\frac{p}{q}(m,n,p,q\in \mathbb{N^{*}};GCD(m,n)=1;GCD(p,q)=1$
$\Rightarrow (\frac{m}{n})^2=x \in \mathbb{N^{*}}$
Mà $GCD(m,n)=1$ suy ra $n=1$
Do đó $x=m^2$
Tương tự ta có $y=p^2$
Từ $(1),(2)$ suy ra $2(\sqrt{x}+\sqrt{y}-1)=\sqrt{xy}$
$\Rightarrow mp=2(m+p-1)$
$\Leftrightarrow (m-2)(p-2)=2$
Từ đây ta tìm được $m=3,p=4$ và $m=4,p=3$
Vậy $(x,y) \in {(9;16),(16;9)}$


 




#715898 Cho p là số nguyên tố; m,n thuộc N* t/m 1+1/2+...+1/p-1=m/n.CMR (p-1)/m

Đã gửi bởi Kim Vu on 23-09-2018 - 09:49 trong Số học

Cho p là số nguyên tố; m,n thuộc N* thỏa mãn: $1+\frac{1}{2}+...+\frac{1}{p-1}=\frac{m}{n}$. CMR: $m\vdots p$.

  

$A=1+\frac{1}{2}+...+\frac{1}{p-2}+\frac{1}{p-1}$

$=(1+\frac{1}{p-1})+(\frac{1}{2}+\frac{1}{p-2})+...+(\frac{1}{\frac{p-1}{2}}+\frac{1}{\frac{p+1}{2}})$
$=p.[\frac{1}{1(p-1)}+\frac{1}{2(p-2)}+...+\frac{1}{\frac{p-1}{2}.\frac{p+1}{2}}]$
Suy ra $A(p-1)!  \vdots  p $   (nhân thêm $(p-1)!$ để đảm bảo số chia là số nguyên)

hay$(p-1)!.\frac{m}{n}\vdots p \Rightarrow m(p-1)!\vdots p$ 
Mặt khác ta có $((p-1)!,p)=1$
Suy ra $m \vdots p$
 




#715872 [TOPIC] Hai bài toán mỗi ngày.

Đã gửi bởi Kim Vu on 22-09-2018 - 18:20 trong Chuyên đề toán THPT

Bài 85: Cho dãy số $(x_n)(n=1,2,...)$ được xác định bởi:

$\left\{\begin{array}{I} x_1=\frac{1}{2}\\ x_{n+1}=x_n-x_n^2+x_n^3-x_n^4+...+x_n^{2007}-x_n^{2008}(n\in \mathbb{N^*})\end{array}\right.$

Tìm $\lim\limits_{n\to +\infty} nx_n$.

Bài 86: Với mỗi số nguyên dương $n$, ta xét hàm số $f_n$ trên $\mathbb{R}$ được xác định bởi :

$f_n(x)=x^{2n}+x^{2n-1}...+x^2+x+1$.

a) Chứng minh rằng hàm số $f_n$ đạt giá trị nhỏ nhất tại một điểm duy nhất.

b) Gọi giá trị nhỏ nhất của hàm số $f_n$ là $S_n$ đạt tại điểm $x_n$. Chứng minh rằng:

i) $S_n>\frac{1}{2}$ với mọi $n$ và không tồn tại số thực $a>\frac{1}{2}$ sao cho $S_n>a$ với mọi $n$.

ii) $(S_n)(n=1,2,...)$ là dãy giảm và $\lim\limits S_n=\frac{1}{2}$.

iii) $\lim S_n=-1$. 

 

Bài 86
a)
Với $x \geq 0 ,f_{n}(x) \geq 1$
Với $x \leq -1 ,f_{n}(x) \geq 1$
Với $x\in (-1;0)$
$f_{n}(x)=\frac{x^{2n+1}-1}{x-1}\\
f'_{n}(x)=\frac{2nx^{2n+1}-(2n+1)x^{2n}+1}{(x-1)^2}$
Xét $h(x)=2nx^{2n+1}-(2n+1)x^{2n}+1\\
h'(x)=2n(2n+1)x^{2n-1}(x-1)>0,\forall x\in (-1;0)$
Lại có $h(0)=1;h(-1)=-4n$ nên $h(0).h(-1)<0$
Suy ra $h(x)$ có nghiệm duy nhất trên $(-1;0)$
Do đó $f'_{n}(x)$ có nghiệm duy nhất trên $(-1;0)$
Lập bảng biến thiên suy ra $f_{n}(x)$ đạt giá trị nhỏ nhất tại một điểm duy nhất
b)

i)    Dễ thấy  $S_{n}=\frac{x_{n}^{2n+1}-1}{x_{n}-1} \geq \frac{1}{2}$  (vì $x\in (-1;0)$)

Giả sử tồn tại số thực $a$ thỏa mãn điều kiện
Ta có:$f_{n}(x)\geq S_{n}>a,\forall n\in N*$
$\Rightarrow \frac{1-x^{2n+1}}{1-x}\geq a,\forall n \in N*$
Do $x\in (-1;0)$ nên khi $n\rightarrow +\infty$ ta có 
$\frac{1}{1-x}>a$ với $x\in (-1;0)$
Cho $x \rightarrow -1^{+}$ ta có $\frac{1}{2} >a$,mâu thuẫn với giả thiết
ii)   Ta có $S_{n+1}=f_{n+1}(x_{n+1})\leq f_{n+1}({x_{n}})$(do $f_{n+1}(x)$ đạt giá trị nhỏ nhất tại điểm duy nhất $x_{n+1}$)
$=f_{n}(x_{n})+x_{n}^{2n+2}+x_{n}^{2n+1} \leq f_{n}(x_{n})=S_{n}(x)$ do $x \in (-1;0)$
Suy ra $(S_{n})$ giảm và bị chặn dưới bởi $\frac{1}{2}$ nên có giới hạn là $b\geq \frac{1}{2}$
Giả sử $b > \frac{1}{2}$ thì $S_{n} >b>\frac{1}{2}$ nên theo câu i) ta có điều mâu thuẫn
Vậy $limS_{n}=b=\frac{1}{2}$
iii)???




 




#715870 $2^{2003}-1$

Đã gửi bởi Kim Vu on 22-09-2018 - 17:13 trong Số học

$2^{2003}-1$  là số nguyên tố hay hợp số, tại sao ? 

Chứng minh $2^{2003}-1 \vdots 4007$ như sau
Theo đinh lí $Fermat$,$2^{4006}-1 \vdots 4007\\
\Leftrightarrow (2^{2003}-1)(2^{2003}+1) \vdots 4007$
mà $GCD (2^{2003}-1;2^{2003}+1)=2$ nên $2^{2003}-1 \vdots 4007$ hoặc $2^{2003}+1 \vdots 4007$
Nếu  $2^{2003}-1 \vdots 4007$ ta có ĐPCM
Nếu $2^{2003}+1 \vdots 4007 \Leftrightarrow 2^{2004} \equiv -2$(mod $4007$)
Suy ra $-2$ là số chính phương modulo $4007$ hay $(\frac{-2}{4007})=1$
Ta có :$(\frac{-2}{4007})=(\frac{2}{4007}).(\frac{-1}{4007})=(-1)^{\frac{4007^2-1}{8}}.(-1)^{\frac{4007-1}{2}}=-1$  
Từ đây ta có điều mâu thuẫn
Vậy $2^{2003}-1 \vdots 4007$ và $2^{2003}-1> 4007$ nên $2^{2003}-1$ là hợp số




#715868 $\sum \frac{a^4}{b^3+c^3}$

Đã gửi bởi Kim Vu on 22-09-2018 - 16:52 trong Bất đẳng thức - Cực trị

Cho $a,b,c>0$

Chứng minh
$\sum \frac{a^4}{b^3+c^3} \geq \frac{a+b+c}{2}$

$\sum \frac{a^4}{b^3+c^3}\geq \frac{a+b+c}{2}\\ \Leftrightarrow \sum \frac{a(a^3+b^3+c^3)-a(b^3+c^3)}{b^3+c^3}\geq \frac{a+b+c}{2} \\ \Leftrightarrow \sum \frac{a(a^3+b^3+c^3)}{b^3+c^3}\geq \frac{3}{2}(a+b+c) \\ \Leftrightarrow \sum \frac{a}{b^3+c^3}\geq \frac{3(a+b+c)}{2(a^3+b^3+c^3)}$
$\sum \frac{a}{b^3+c^3}=\sum \frac{a^2}{a(b^3+c^3)} \geq  \frac{(a+b+c)^2}{\sum a(b^3+c^3)}$(Cauchy-Schwarz)
Cần chứng minh $2(a+b+c)(a^3+b^3+c^3) \geq 3\sum a(b^3+c^3)$
$\Leftrightarrow 2(a^4+b^4+c^4) \geq ab^3+a^3b+bc^3+b^3c+ca^3+c^3a$
Điều này đúng do $a^4+b^4 \geq ab^3+a^3b$




#715798 Đề thi chọn đội tuyển HSG dự thi Quốc gia Quảng Trị 2017-2018

Đã gửi bởi Kim Vu on 20-09-2018 - 22:22 trong Thi HSG cấp Tỉnh, Thành phố. Olympic 30-4. Đề thi và kiểm tra đội tuyển các cấp.

Lời giải khác cho câu hình ngày 1

$DF\cap AC=M$
Tứ giác $AFDH$ nội tiếp suy ra $\widehat{MDE}=\widehat{EAC}=\widehat{ACE}$ nên tg $MDCE$ nội tiếp $(C)$
$AF\cap (C)=S$
$\widehat{AMS}=\widehat{AEC}=\widehat{BAC}$ nên $MS \parallel AB$
$\widehat{ADB}=\widehat{ACB}=\widehat{DCE}-90=90-\widehat{DSE}=\widehat{ADS}$ nên $D,B,S$ thẳng hàng
$DF\cap AB=P$
$AD,AS$ là pg trong và ngoài $\widehat{BAC}$ nên $(AC,AB,AF,AD)=-1$
Do đó $(MPFD)=-1$
suy ra $B(MPFD)=-1$ hay $B(MAFS)=-1$
Gọi $AD\cap SM=T$ ,do $\bigtriangleup SAT$ vuông ,$MS=MA$ nên M là trung điểm $ST$
Lại có $BA \parallel ST$
nên $B(MATS)=-1=B(MAFS)$
Suy ra $B,F,T$ thẳng hàng
Từ $\triangle AMS \sim \triangle CEA(g.g) ;\triangle AMF \sim \triangle CEH$(g.g)  suy ra $\frac{AH}{HC}=\frac{SF}{FA}$
Mặt khác $\frac{SF}{FA}=\frac{TF}{FB}=\frac{AK}{KB}$
 $\Rightarrow \frac{AH}{HC}=\frac{AK}{KB}$ 
Suy ra $HK\parallel BC$ 

Geo.png

 




#715782 CMR $f(7)$ không là số chính phương

Đã gửi bởi Kim Vu on 20-09-2018 - 19:11 trong Số học

Giả sử $f(7)$ là số chính phương thì $f(7)$ chia $6$ dư $0;1;3;4$

Mặt khác $f(7)-f(1) \vdots 6$ nên $f(7)$ chia $6$ dư $2$ vô lí




#715713 Đề thi chọn đội tuyển trường PTNK năm 2018

Đã gửi bởi Kim Vu on 18-09-2018 - 22:40 trong Thi HSG cấp Tỉnh, Thành phố. Olympic 30-4. Đề thi và kiểm tra đội tuyển các cấp.

Bài 3
a)Với $p=5$
$x^3+x+5=y^2$
Nếu $x$ chia 4 dư 1,2,3 thì $y^2$ chia 4 dư 3 nên không thỏa mãn
Nếu $x$ chia hết cho 4
PT$\Leftrightarrow (x+3)(x^2-3x+10)=y^2+25=y^2+5^2$
$x+3$ có ít nhất một ước nguyên tố $h=4k+3$ nên $y^2+5^2\vdots h=4k+3$
Theo bổ đề về tổng bình phương và số nguyên tố,ta có $y\vdots h;5\vdots h \Rightarrow h=5$(vô lí)
Vậy với p=5 PT không có nghiệm nguyên
Với $p=13$,PT có nghiệm một nghiệm là $x=4;y=9$
Vậy $p=13$ là giá trị nhỏ nhất cần tìm
 




#715680 $BC,QR$ và tiếp tuyến tại $P$ của $(O)$ đồng quy.

Đã gửi bởi Kim Vu on 17-09-2018 - 22:57 trong Hình học

Gọi giao điểm điểm tiếp tại $P$ của $(O)$ với $QR $ là $I$
Ta sẽ chứng minh $E,D,I$ thẳng hàng
Từ giả thiết suy ra $BE=DC;EC=BD$
$ER.EA=EB.EC;DQ.DA=DB.DC$ nên $ER.EA=DQ.DA \rightarrow \frac{DQ}{ER}=\frac{AE}{AD}$
Mặt khác $\frac{AE}{AD}=\frac{sin\widehat{MAD}}{sin\widehat{EAM}}=\frac{PQ}{PR}$
Và $\frac{IR}{IQ}=(\frac{PR}{PQ})^2$
nên ta có:
$\frac{EA}{ER}.\frac{DQ}{DA}.\frac{IR}{IQ}=\frac{PQ}{PR}.\frac{PQ}{PR}.(\frac{PR}{PQ})^2=1$
Suy ra ĐPCM

 




#715672 Chứng minh rằng BC, B1C1, B2C2 đồng quy.

Đã gửi bởi Kim Vu on 17-09-2018 - 20:51 trong Hình học

Cho tam giác ABC không cân tại A. Đường tròn nội tiếp (I) tiếp xúc với BC tại D. Điểm M thuộc đoạn AD. Các đường thẳng MB, MC theo thứ tự cắt (I) tại B1, B2, C1, C2 (BB1<BB2 , CC1<CC2).

Chứng minh rằng BC, B1C1, B2C2 đồng quy.

Gọi $E,F$ là tiếp điểm của $(I)$ và $AC,AB$
$AD$ cắt $(I)$ tại $K$
Tiếp tuyến $(I)$ tại $K,D$ giao tại $S$
DB1KC1 là tứ giác điều hòa nên tiếp tuyến $(I)$ tại $K,D$ cắt nhau trên $EF$ hay $S,E,F$ thẳng hàng
Suy ra $(SDBC)=-1$
Gọi giao của SBvới $AD,(I)$ là L và C2
$KD$ là đường đối cực của $S$ đối với $(I)$ nên (SLB1C2)=-1
Do đó (SDBC)=(SLB1C2)=-1
Suy ra LD,BB1,CC2 đồng quy nên Ctrùng C1
Như vậy ta đã chứng minh B1Cđi qua $S$
Tương tự B2C2 đi qua $S$  :icon1:
 
gd.png



 




#715660 Bài tập thi HSG12 QUẢNG NGÃI

Đã gửi bởi Kim Vu on 17-09-2018 - 17:05 trong Số học

BÀI TẬP : Tìm tất cả các ước nguyên tố n của $A=2^{2017}-1$ biết $n<9000$

$n$ là ước nguyên tố bất kì của $A$ nhỏ hơn 9000
Gọi h là cấp của 2 modulo n;ta có ordn(2)$=h \rightarrow 2^h-1 \vdots n$
$2^{2017}-1 \vdots n;2^{n-1}-1 \vdots n \rightarrow h$ là $UC(2017;n-1)$
$\rightarrow h \in \left \{ 1;2017 \right \}$
TH1:$h=1$ thì $2-1 \vdots n$(vô lí)
TH2:$h=2017 \rightarrow n-1 \vdots 2017 \rightarrow$ mọi UNT của $A$ đều chia 2017 dư 1 
Các số tự nhiên nhỏ hơn 9000 và chia 2017 dư 1 là 1;2018;4035;5052;8069
Chỉ có 8069 là số nguyên tố
Tuy nhiên nếu nếu $A\vdots 8069$ thì $2^{2017}\equiv 1(mod 8069)$
$ \Rightarrow 2^{2018} \equiv 2(mod 8069) \Rightarrow 2$ là số chính phương $modulo$ $8069$
 $\Rightarrow (\frac{2}{8069})=1$ 
Mặt khác $(\frac{2}{8069})=(-1)^{\frac{8069^2-1}{8}}=-1$(Mâu thuẫn)
Suy ra 8069 không là ước nguyên tố của $A$

Vậy $A$ không có UNT nhỏ hơn 9000
 




#715613 Cho P(x) thuộc R[x] degP>=1 có m nghiệm Cm Q(x)=(x^2+1)P(x)+P'(x) có...

Đã gửi bởi Kim Vu on 16-09-2018 - 12:38 trong Đa thức

Cho P(x) thuộc R[x] degP>=1 có m nghiệm Cm Q(x)=(x^2+1)P(x)+P'(x) có ít nhất 1 nghiệm

Xét $H(x)=e^{\frac{x^3}{3}+x}.P(x)$
$H'(x)=e^{\frac{x^3}{3}+x}((x^2+1)P(x)+P'(x))=e^{\frac{x^3}{3}+x}Q(x)$

TH1:$m=1$.

Nếu $P(x)$ bậc lẻ thì $Q(x)$ bậc lẻ nên có ít nhất 1 nghiệm
Nếu $P(x)$ bậc chẵn,$P(x)$ có 1 nghiệm là $a$ thì $\frac{P(x)}{x-a}$ bậc lẻ nên có ít nhất một nghiệm
 suy ra $P(x)$ có ít nhất 2 nghiệm(có thể trùng nhau)(vô lí)
TH2:$m \geq 2$
Do $H(x)$ có m nghiệm ,nếu có ít nhất 2 nghiệm phân biệt theo định lý Rolle $H'(x)$ có ít nhất 1 nghiệm nên $Q(x)$ có ít nhất 1 nghiệm
Nếu $H(x)$ chỉ có nghiệm bội m thì $H'(x)$ có nghiệm bội $m-1$ suy ra $Q(x)$ có ít nhất 1 nghiệm




#715612 Tìm tất cả các cặp (x,y) thỏa mãn

Đã gửi bởi Kim Vu on 16-09-2018 - 12:11 trong Số học

Tìm tất cả các cặp số nguyên (x,y) thỏa mãn:

x4+x3+x2+2x=y(y+1)

$4x^4+4x^3+4x^2+8x+1=(2y+1)^2$
Kẹp giữa $(2x^2+x)^2$ và $(2x^2+x+1)^2$




#715598 [TOPIC] Hai bài toán mỗi ngày.

Đã gửi bởi Kim Vu on 16-09-2018 - 01:35 trong Chuyên đề toán THPT

Bài 73: Chứng minh rằng nếu các số dương $a,b,c$ thỏa mãn: $abc=1$ thì $\frac{a}{\sqrt{8c^3+1}}+\frac{b}{\sqrt{8a^3+1}}+\frac{c}{\sqrt{8b^3+1}}\ge 1$.

Bài 74: Xác định tất cả các đa thức với hệ số thực $P(x),Q(x)$ và $R(x)$ thỏa mãn điều kiện: $\sqrt{P(x)}-\sqrt{Q(x)}=R(x)$ với mọi số thực $x$.

Bài 73
$A=\sum \frac{a}{\sqrt{8c^3+1}}=\sum \frac{a}{\sqrt{(2c+1)(4c^2-2c+1)}}\geq \sum \frac{a}{2c^2+1}$
Đặt $a=\frac{x}{y};b=\frac{y}{z};c=\frac{z}{x}$
$ \sum \frac{a}{2c^2+1} = \sum \frac{x^3}{2z^2y+yx^2}=\sum \frac{x^4}{2z^2yx+yx^3} \geq \frac{(x^2+y^2+z^2)^2}{2xyz(x+y+z)+x^3y+y^3z+z^3x}$
$3x^4+y^4 \geq 4x^3y  \\3y^4+z^4 \geq 4y^3z  \\3z^4+x^4 \geq 4z^3y \\\rightarrow x^4+y^4+z^4 \geq x^3y+y^3z+z^3x$
$(xy)^2+(yz)^2+(zx)^2\geq xyz(x+y+z)$
Do đó ta có $\frac{(x^2+y^2+z^2)^2}{2xyz(x+y+z)+x^3y+y^3z+z^3x} \geq 1$




#715578 Cho $(u_{n}):u_{n+1}-2u_{n}+u_{n+1...

Đã gửi bởi Kim Vu on 15-09-2018 - 20:36 trong Dãy số - Giới hạn

Cho $(u_{n}):u_{n+1}-2u_{n}+u_{n+1}=K$, $K$ là hằng số (Với mỗi $n\geq 2$). Tính $lim\frac{u_{n}}{n^{2}}$

Chắc hẳn đề là $u_{n+1}-2u_{n}+u_{n-1}=K$
Sử dụng định lí Stolz liên tiếp 
$lim\frac{u_{n+1}-2u_{n}+u_{n-1}}{(2n+1)-(2n-1)}=\frac{K}{2}\Rightarrow lim\frac{u_{n}-u_{n-1}}{n^2-(n-1)^2} =lim \frac{u_{n}-u_{n-1}}{2n-1}=\frac{K}{2}\Rightarrow lim\frac{u_{n}}{n^2}=\frac{K}{2}$